Storing Cookies (See : http://ec.europa.eu/ipg/basics/legal/cookies/index_en.htm ) help us to bring you our services at overunity.com . If you use this website and our services you declare yourself okay with using cookies .More Infos here:
https://overunity.com/5553/privacy-policy/
If you do not agree with storing cookies, please LEAVE this website now. From the 25th of May 2018, every existing user has to accept the GDPR agreement at first login. If a user is unwilling to accept the GDPR, he should email us and request to erase his account. Many thanks for your understanding

User Menu

Custom Search

Author Topic: The Lorentz Force is an APPROXIMATION  (Read 14441 times)

kmarinas86

  • Full Member
  • ***
  • Posts: 156
    • YouTube - kmarinas86's Channel
The Lorentz Force is an APPROXIMATION
« on: January 13, 2014, 09:41:04 AM »
This is key to understanding various energetic anomalies involving unusual
electromagnetic devices, including Joseph Newman's energy machine and Bruce DePalma's N-
machine.
https://web.archive.org/web/20140113061513/http://www.scielo.br/scielo.php?pid=S1806-11172012000200006&script=sci_arttext

// "
4. Conclusions
We have shown in the present work that, by using the vectorial identities, Eqs. (9)-(11),
it is possible to obtain directly from Maxwell's equations the momentum balance equation,
identifying in it a force density that we have called Maxwell's force density. Robinson's
criticism [9] that this type of deduction is a mere identity does not constitute a
demerit; otherwise we would have to blame also the continuity equation of charge and
current, since it also results from an identity derived from Maxwell's equations.
It is also clear from this deduction that the usual Lorentz force is an approximation for
very small charges and corresponding currents, which permits to neglect the self fields.
Therefore the generalized Lorentz force, including total fields, is implicit in
Maxwell's equations.
The inclusion of total fields is essential to the study of
radiation reaction. On the other hand such a general result like this must be tested with
particular models and experiments. In this sense is remarkable that the Helmholtz force
density for fluids is contained, as a particular case, in the Maxwell balance equation.
This is a relevant result not appearing in the literature. A final point, relevant for
teaching, is the use of vector identities, not all familiar to students that permit a
straightforward deduction, avoiding in this way the tricky deductions with components
that obscure the deduction and make it repellent to the reader.
" //

The significance of this:
1) The current of a conductor does not act directly on an external magnetic field.
2) The magnetic field of current of a conductor does act directly on an external
magnetic field.
The Lorentz force calculates the force on a wire based on the field strength through that
wire. However, the magnetic field of the current of the wire extends far beyond the
limits of the wire. How this matters is based on the fact that the magnetic field of a
magnet is a function of position. There are different pairs of points on the magnetic
field of a magnet where the magnetic field strength points are mutually opposite in
direction. Since the magnetic field of a current extends to infinity and the magnetic
field of a magnet extends to infinity, then their forces are distributed throughout all
of space. Only a calculation of the forces over all of space, as the spatial integral of
force density, using Maxwell's equations, should give the correct answer. The Lorentz
force is thus not only redundant and can be accounted for by applying a space integral,
it is also inaccurate.

The only reason why the Lorentz force should be used is when it doesn't make a
difference. The reasons why it is used is probably due to the ease of calculating
a single path integral vs. the difficulty of calculating a four-integral that combines a
path integral with a space integral.

In the case of the Newman machine:
There is a range of angles close to top dead center (TDC) where the induced magnetic
field due to rotation of the magnet field of the magnet through the conducting coil is
actually reverse of what would slow down the magnet. To see why, visualize the magnetic
field around a magnet. Then imagine conductors passing through this field.

https://sites.google.com/site/kmarinas86/energy/newman-machine/Newman61.png

As might be expected in any motor, some of the turns of the coil will have induced in
them voltages opposed to that of other turns in the same coil. If we position the magnet
so that more of the lines that cut through the conductor induce current in the
other direction, the induced magnetic field of the coil will actually repel the
magnet away from top dead center if the magnet is rotating from top dead center toward
the coil, while in the other case, where the magnet moves toward top dead center when it
is already close to it, the induced magnetic field will actually pull the magnet toward top
dead center. The problem with this is that this effect only occurs in a small angle, so
it becomes important to reuse this energy for other parts of the rotation. In addition,
this effect turns out to be very weak, as the peripheral field is less energy dense. So
over the course of a full rotation, the back-emf tends to dwarf the forward emf.

Below is an image of the Newman machine slightly off to top dead center:

https://sites.google.com/site/kmarinas86/energy/newman-machine/Newman70.png

As I stated, the magnetic field of the magnet does not act directly on a current. It acts
on the magnetic field of that current, and because it may vary in direction in intensity
and direction throughout space, the Lorentz force is false and at best an
approximation. However, a changing magnetic field produces an electric field. It is the
change of this magnetic field (related to the time derivative of the magnetic vector
potential) which produces the induced current. A function is not generally the same as
its time derivative. The forces that drive the relative motion between the magnet and
the wire are technically not the same as those forces that drive the current. The
magnetic vector potential is technically what drives the induced emf. Look at the fact of
momentum conservation. For every force is an equal and opposite force. So knowing that
the force between the magnet and the conductor is essentially at a right angle to the
acceleration of charge due to induced emf, this means the forces between the magnet and
the wire do not account for what happens in the wire itself. Why? Total momentum
conservation requires that all forces must balance. If your problem does not conserve
momentum, then either you did the problem wrong, or there is something else that
exists which balances this equation. In this case it is the magnetic vector potential of
a line of magnetic flux; this does line up on the same axis as the induced current. The
force against (or in this case, with) the rotating magnet, a force which is tangential to
the shaft rotation, cannot account for the force which is parallel to the current, as they
are essentially perpendicular. So the force that drives the current exists on what is
essentially a different force channel than what drives the force between the
magnetic field of the magnet and the magnetic field of the current, a force that acts
perpendicularly to both the magnetic field and the current which produces it.
Below is an image of the Newman machine version with the magnet outside:

https://sites.google.com/site/kmarinas86/energy/newman-machine/Newman43.png

As far as the N-machine is concerned, you must realize that the Lorentz force is
essentially false
(and so is the Laplace force). The key is in finding that the
magnetic field that is parallel with the shaft is not being rearranged as it spins due to
the symmetry of the magnet, so essentially there is no force between it and the magnetic
field of the wire that would cause it to speed up or slow down. The magnetic vector
potential, which is reflected off the electrons and thereby produces a current acts on a
completely separate axis and cannot slow down the rotation of the magnet.

MarkE

  • Hero Member
  • *****
  • Posts: 6830
Re: The Lorentz Force is an APPROXIMATION
« Reply #1 on: January 13, 2014, 10:09:35 AM »
Kmarinas, Joseph Newman's energy machine was subjected to very detailed tests by the NBS, now NIST, who found that it did not generate surplus energy.  No one has shown surplus energy from Bruce DePalma's machine either.  The Lorentz Force has been subjected to very careful evaluation by NBS, and Ampere balances were used for many years to calibrate weights.  If the Lorentz force varied even by a small amount, those weight calibrations would be off.

If you wish to argue that the Lorentz Force is inaccurate, then you should propose and execute an experiment that would show that.

Marsing

  • Sr. Member
  • ****
  • Posts: 300
Re: The Lorentz Force is an APPROXIMATION
« Reply #2 on: January 13, 2014, 10:12:37 AM »


approximation!!,
if everyone accept this...   

PiCéd

  • Full Member
  • ***
  • Posts: 172
Re: The Lorentz Force is an APPROXIMATION
« Reply #3 on: January 13, 2014, 11:34:07 AM »
If the Lorentz for is conservative then somebody must certainly have a new equation to prove that.
I realy think that this force is conservative but to do that his equation must be 0.

kmarinas86

  • Full Member
  • ***
  • Posts: 156
    • YouTube - kmarinas86's Channel
Re: The Lorentz Force is an APPROXIMATION
« Reply #4 on: January 13, 2014, 10:23:39 PM »
You might be interested in the following pictures. These are also posted at http://www.energeticforum.com/renewable-energy/16504-snap-wrap-bracelet-visualization-magnetic-induction.html.

To be more precise about the third image (http://www.overunity.com/14200/the-lorentz-force-is-an-approximation/dlattach/attach/131982/image//), the right-hand example does not violate the Lorentz force per se, as the rules for electromagnetic induction used in the example imply the Lorentz force (which is exerted in the direction of charge acceleration due to relative motion of the magnetic field through the conductor). But what it does violate is Fleming's right-hand rule for generators. The force between the magnet and the current it induces is the same you would expect if the system were being driven as a motor. This is possible due to the special placement of the magnet and the windings (very untypical).

kmarinas86

  • Full Member
  • ***
  • Posts: 156
    • YouTube - kmarinas86's Channel
Re: The Lorentz Force is an APPROXIMATION
« Reply #5 on: January 14, 2014, 03:07:50 AM »
You might be interested in the following pictures. These are also posted at http://www.energeticforum.com/renewable-energy/16504-snap-wrap-bracelet-visualization-magnetic-induction.html.

To be more precise about the third image (http://www.overunity.com/14200/the-lorentz-force-is-an-approximation/dlattach/attach/131982/image//), the right-hand example does not violate the Lorentz force per se, as the rules for electromagnetic induction used in the example imply the Lorentz force (which is exerted in the direction of charge acceleration due to relative motion of the magnetic field through the conductor). But what it does violate is Fleming's right-hand rule for generators. The force between the magnet and the current it induces is the same you would expect if the system were being driven as a motor. This is possible due to the special placement of the magnet and the windings (very untypical).

I attached some JPG versions to this post for those who can't view PNG images.

kmarinas86

  • Full Member
  • ***
  • Posts: 156
    • YouTube - kmarinas86's Channel
Re: The Lorentz Force is an APPROXIMATION
« Reply #6 on: November 30, 2014, 07:15:45 AM »
Here is another article casting doubt on the idea that the Lorentz Force is a necessary "add-on" to Maxwell's Equations.

http://arxiv.org/pdf/1211.6072.pdf

Quote from: A fields only version of the Lorentz Force Law: Particles replaced by their fields
We show that the Lorentz force law, F^L_1=q_1(E+v_1xB) being the charge on particle 1 interacting with the electromagnetic fields due to all other particles, can be written in a pure field form F^L_1=-\nabla_1 U^{EM}. In this expression U^{EM} is the total electromagnetic energy of the system of particle 1 and all other particles. In deriving this result we review the old but not widely known results that Maxwell's equations follow uniquely from Special Relativity, and that the Lorentz force law follows from applying Hamilton's variational principle to this result.
For a two particle system, the standard view is that the electromagnetic force on particle 1 is the result of the charge of particle 1 interacting locally with the field of particle 2, and conversely. Both charges 1 and 2, and fields 1 and 2 are needed. In our approach, the fields of all particles contribute to the electromagnetic interaction everywhere, over all of space. The charges of the particles do not enter the theory except incidentally, via Gauss's law. This has novel interpretational consequences. In particular, it allows a charged particle to be replaced by its electric and magnetic fields, much as a particle in quantum mechanics is replaced by its complex valued wavefunction.

The last sentence of the above quotation is most interesting. In essence, it suggests that an electric charge is the one and the same as its field. "Field" and "charge" are not separate. They are part and parcel. Disturbances within this field can travel at the speed of light, but that does not mean they have to start from the center. However, since most of the energy of this field exists close to the "center" of this field, where the field is strongest, most of the disturbance tends to propagate from there, giving the illusion of a localized "point" charge having a "unique" position wherefrom the time-retarded field of a charge may emanate. Opposed to the idea that the time-retarded field of a charge emerges solely from the "one-and-only-point" where this charge exists, the idea here is that the time-retarded field is due to the change of a charge's field at every point in space. Any apparent "faster-than-light" phenomenon would therefore seem to be the result of charges having the bulk of their interactions occur at weaker fields far away from their centers, or in a phrase: "far-field to far-field coupling". It is only apparent because the charge is actually the same thing as its field and is in essence already there in a physical sense as an electric flux pervading an infinite volume rather than as a probabilistic wave function of some "point charge" which "may or may not" be there. It should also be noted that this explanation rather elegantly resolves the particle-wave duality without violating the common sense notion of the existence of a concrete reality (see "pilot wave theory" and "GUT-CP" for supporting evidence of such a concrete reality).

Special attention should be paid to the fact that the energy density of a field is proportional to the square of the field intensity (quadratic), even though the fields themselves are additive (linear). We can think of the self-energy density of the charges as being a linear sum, while the local "interaction energy density", as the article calls it, is something that adds to or subtracts from the local self-energy density, which simply put is the result of mutual capacitance and mutual inductance, both of which can be (independently) positive or negative. For example, the braking action due to Lenz's law is due to inducing a current whose magnetic moment is opposed to the change of an external magnetic field, which can be likened as a reaction to the change of the source current for such a magnetic field (even if the external magnetic field is from a permanent magnet); this example is the result of a negative mutual inductance. It could be said that all capacitance and inductance originates from mutual interaction of the quintessential substance responsible for the vector potential.

According to the paper at:
http://adsabs.harvard.edu/full/1956ApJ...123..508B

Rotating a magnet around its principal magnetic axis (which rotates the azimuthally-directed vector potential azimuthally around the axis of the magnet) results effectively in what is, as per Alfvén's formula (equation 1 in that paper), an inverse square electric field (if and only if charge motion were constrained along field lines). Well, what do you know, in Newman's theory the "charges" or Gyroscopic Particles which are responsible for magnetic fields also happen to travel along magnetic field lines.

On the last page of that paper, the author states:

Quote from: George Backus
"Leverett Davis (1947, 1948) obtains formulae ( 8) and (9) for a uniformly magnetized sphere rotating in vacuo and shows that if the space surrounding the sphere contains charged particles constrained to move along the magnetic field lines of force, then a charge density will accumulate which will provide exactly the electric field of formula (1) predicted by Alfvén."

[Fun trivia: Joseph Newman's mother had her name changed to Margaritte Davis. Funny how similar that name is to Leverett Davis! Anyway....]

Well, guess what? The orbitsphere as per the Mills model of hydrogen atom has a uniform interior magnetic field! Noting that the orbitsphere is mathematically derived by convolving "Current Vector Fields" around an axis (see "BECVF" and "OCVF"), it should be possible that the precession of a spinning magnet on one or more axes could smooth out any non-uniformity of the intensity of the electric field induced by rotating magnetic fields with respect to angle, resulting in a spherically uniform electric field that drops with the square of the distance. In other words, electric charge is due to spinning magnets, rather than the opposite.

Also interesting is the fact that Mills' model for the free electron is a spinning disk where the translational velocity and the spin velocity are equivalent and perpendicular, while in Newman's model for the magnetic field, the Gyroscopic Particles making up the field move at the speed of light and spin at the speed of light (again perpendicular to the propagation). In short, the spin axis is aligned with the translational motion. This is known as helicity.

In Mills' model of the free electron, the radius of the free electron is inverse to the velocity of the free electron (source: http://www.blacklightpower.com/wp-content/uploads/theory/Mathematica/FreeElectron.pdf). If you were to divide Mills' disk into rings whose velocity varied sinusoidally with mean velocity, correction of each ring's size would result in a torus with the inner rings leading in velocity and outer rings lagging in velocity. The result would be a charged torus, something which is described by Ruggero Maria Santilli's concept of "magnecular" bonds which are responsible for "magnecules".

Using Mills' model, if you combine the magnetic field due to the spin of the electron with the magnetic field due to the translation of the electron, the result is a helical magnetic field, which very likely means a helical vector potential as well. Magnetic helicity is said to be conserved (source: http://en.wikipedia.org/wiki/Magnetic_helicity) and has units of squared magnetic flux. Because of this, it can be said that is has units of energy * inductance. As for momentum, if can be transferred without any final barriers from charge to charge in infinitesimal quantities, the simultaneous conservation of momentum, charge, and vector potential implies that charge (found in the denominator) exists in rational multiples (which is equivalent to the quantization of charge). Conservation of vector potential has been a basis for the concept of vector potential waves (source: http://redshift.vif.com/JournalFiles/Pre2001/V07NO1PDF/V07N1KUL.pdf).

As for charge itself:
http://physics.stackexchange.com/questions/6581/what-is-the-electric-field-generated-by-a-spinning-magnet

It is perhaps of no coincidence that Mr. "MagnetVortex" (a.k.a. Al) was quite successful in capturing infrared footage of UFOs as I believe witnessing UFOs is a privilege for those who have done, or will do, important things in assisting the disclosure of very important messages.

Going back to the first article quoted in this post, here is my favorite paragraph:

Quote from: Philip H. Butler, Niels G. Gresnigt, Martin B. van der Mark, Peter F. Renaud
Our new approach will require a re-interpretation of the usual statements that electromagnetic fields do not interact with other electromagnetic fields but only with charges and currents (for an example see page 226 of reference [3]).

This statement by the above quoted scientists is in harmony with Joseph Newman's assertion that magnetic fields act directly on each other and is the medium which conveys force to the charges, which is opposed to the notion that charge interaction is determined only by the strength of the field at the (point) charge.

Postscript:
* My guess is that if you use the vector potential instead of the magnetic field B, you can still use the local value of field on the charge to get the correct interaction. The changing vector potential can exist even where the curl of the vector potential is 0. This is also the answer to how a transformer really works. If you consider that scientists consider the Lorentz force more fundamental than the Faraday flux rule, at some point a (non-physicist) electrical engineer has to wonder how the Lorentz force could possibly explain how an ideal iron core with no flux leakage can induce a current. This is why they will often rely on the Faraday flux rule instead of the Lorentz force when explaining transformer induction. Use of the vector potential solves this problem.
* I have refined my ideas on what it would take to take advantage of this new discovery. It appears that an S-shaped coil (with a specific three-dimensional geometry in mind) could potentially work without a commutator, battery, capacitor, or any sensitive solid state electronic components. "Lenz' Law" only applies when the velocity of negative/positive charge accumulated due to a changing vector over time is induced in the same/opposite direction as the derivative (rate change) of the vector potential with respect to time. In an "S-Coil" this is reversed at certain segments of the coils, whose length can be optimized as desired. To obtain this with non-trivial results requires that electric scalar potential of charges be changing (as it does when a wave of electrical impulse travels down a wire) so as to do "work" on the charge against the derivative of vector potential (i.e. the derivative of the electrical potential (units of V/s) "works" on the derivative of the vector potential (units of V/m)).
** The process by which energy is extracted from the electrical scalar potential should be analogous to how a Viktor Schauberger's Vortex Engine can extract energy from ambient static pressure, that is to say by manipulating the particles initially under static pressure into a non-isotropic flow function via entrainment in order to develop "dynamic pressure" whose P*V demonstrates excess net of losses, exactly as hurricanes and tornados do when their local static pressure drops dramatically (as well as what happens when "canned air" is quickly depressurized). The concept of "thermodynamic (i.e. static) temperature", which is so much the basis for the 2nd law, fails as soon as you have vorticity in the fluid in a way which violates the assumption of a Maxwell-Boltzmann (or similar) distribution of particle states. This becomes significant as a percentage of input power into a fluid only when you reach significant velocity close to the speed of waves which propagate the medium (in this case the speed of sound), to the point where it is difficult to engineer a solid, reliable device.

It is possible to come up with a much more readily engineerable device that takes advantage of the ambient electric scalar potential as opposed to the ambient static pressure-volume energy of a gas:

https://www.disqus.com/home/discussion/peswiki/directorysyairs_replication_of_no_back_emf_generator/#comment-1714110485

Quote from: kmarinas86
One major key is understanding that while the emf [in volts] induced into a closed loop by a magnetic field is not uniform along its path [i.e. the distribution of V/m as derived from v x B], the current is.

For illustration:
* Take a letter-sized piece of paper and cut out a 2" or 5 cm thick strip (depending on your preference).
* With this strip of paper, draw four arrows, one across within each margin, as close as to the edge of the paper as possible in the same rotating direction (i.e. clockwise or counter-clockwise).
* For clarity, add additional chevrons (or "arrowarms") on the long arrows so you will see the direction of each arrow even when you cannot see the arrowheads.
* Do the same on the other side of the paper but in the opposite rotating direction.
* Now make two creases in the paper in a proportion of pi/2:1:pi/2 dividing the long side (11" in the U.S.) (so approximately 4 1/6", 2 2/3", 4 1/6").
* Curl the pi/2 portions so they now form an S shape, with both pi/2 pieces having the curvature of a semi-circle, while the center piece of proportion remains flat.
* Now cut out a rectangle within the strip, leaving a 0.5" margin containing the arrows you drew.

After completing the above steps, you have created paper model of an "S-coil".

Consider the following:
* Now compare short edges (2" or 5 cm) and the flat segment in the middle piece (2 2/3" in the U.S.).
* You will notice that the arrows on the short edges and arrows in the middle piece run in opposite rotating directions in the plane which intersects both [the "central plane"].
* If you rotated a bar magnet inside the center of the "S-coil" so that the movement of the field lines would run perpendicular to the short edges but parallel to the curved pieces, you will realize that:
** Most of the EMF will be induced on the short (2" or 5 cm) edges.
** The rest of the EMF will be induced on the flat (2 2/3") middle piece.
** No EMF will be induced on the curved (4 1/6") portions.
** Most of the current in the "central plane" runs in the direction of the flat (2 2/3") middle piece (because 2 2/3" > 2").
* If you imagine the magnetic field from current flowing with the arrows, you will notice that:
** The magnetic field from the curved pieces will intersect the "central plane" at an oblique angle, reducing the effective field strength through that plane.
** The magnetic field from the non-curved segments will intersect the "central plane" perpendicularly, thus substantially contributing to the effective field strength through that plane.

Consider the moment when the magnet rotates through the "central plane":
* If the magnetic force were solely due to the (qv x B) force on the wire, it is easy to see that rotating the bar magnet would induce an emf in the short (2" or 5 cm) segments, generating a current which slows down the bar magnet.
* However, there is also a magnetic force on the bar magnet from the current in the flat middle piece, even though minimal EMF is induced into it by the bar magnet; this is possible to due the unequal distribution between the induced field on each wire element and the resultant current running through each wire element.
* The direction the magnet will turn is ultimately determined by effective "amp turns" acting on the magnet, and here the current in the flat middle pieces wins because the magnetic field from the curved pi/2 segments is largely oblique (and as well as cancelling) at the "central plane", while the current from the short (2" or 5 cm) edges clearly produces less magnetic flux through the central plane due to having less "amp x meters".

http://en.wikipedia.org/wiki/Hamiltonian_mechanics
"[T]he Lagrangian of a non-relativistic classical particle in an electromagnetic field is (in SI Units):"
http://upload.wikimedia.org/math/8/5/0/8502f581770b120147a669354563a344.png

The 1st term on the RHS [1/2 m |v|^2] is the kinetic energy. [m = "mass"; v = "velocity"]
The 2nd term on the RHS [+ e v·A] is the magnetic potential energy. [e = "charge"; v="velocity"; A="magnetic vector potential"]
The 3rd term on the RHS [- eΦ] is the electric potential energy. [e = "charge"; Φ="electric potential"]

Now consider what happens in the coil:
* Along the wire are differential path elements (s') along ("along" = "directed with") the wire.
* dA/dt = -1 * E_induced,q [Where: dA/dt is the total derivative of the magnetic vector potential with respect to time (perspective of a moving charge), and the E_induced,q is the induced electric field from the rotating magnet (in q's frame of reference)]
* ∂A/∂t = -1 * E_induced,lab [Where: ∂A/∂t is the partial derivative of the magnetic vector potential with respect to time (perspective of the lab's inertial frame), and the E_induced,lab is the induced electric field from the rotating magnet (in the lab's frame of reference)]
* The vector potential [A] of the magnet wraps around the axis of the magnet in the same direction that a moving positive charge would generating the same field.
* The vector potential [A] generates an electric field which opposes it, and since electrons are accelerated in the opposite direction of the electric field, the induced electron flow follows the change of the magnetic vector potential, which resists the change of the magnetic vector potential.
* Because the magnitude of the vector potential decreases as angle away from the magnetic moment [m] decreases, the derivative of the vector potential (= -1 * E_induced,lab) intersects with the induced electron current (v_induced·(- E_induced),lab > 0; v_induced·∂A/∂t > 0) converting magnetic energy (Δ(e v·A) < 0) at (magnetic) pressure into some other form of energy at the short (2" or 5 cm) edges and intersects against the induced electron current (v_induced·(- E_induced),lab > 0; v_induced·∂A/∂t > 0) converting some form of energy into magnetic energy (Δ(e v·A) > 0) at (magnetic) pressure at the flat (2 2/3") middle piece, where "intersects with" means the angle is between -90 and +90 degrees (a positive dot product) and "intersects against" means the angle is between +90 and +270 (a negative dot product) [Note: we are considering the electron current, not conventional current]
* Force is a rate change of momentum (with respect to time). The force times a charge-to-mass ratio gives [e * a] or a product of charge and acceleration. The integration of this value with respect to time (assuming constant charge e) is [e * v]. The dot product between [e * v] and A gives the magnetic potential energy. If a force (e ∇Φ) is transferred from one charge (q_1) to another (q_2) via the electric field (∇Φ), the magnetic potential energy can be made to increase depending on how the vector potential intersects each one. Note that in principle, this requires a minimum of 3 charges, where one charge represents the source of the external magnetic field (with its spatially-varying vector potential), and the two remaining charges are those two being considered in this scenario which are subject to different vector potentials. It follows that this energy (if a positive change) must either come from kinetic energy, electrical potential energy, or something else:
** If it comes from kinetic energy, then the current in the S-coil will have an anomalous resistance in the flat middle portion where the induced electron current is flowing against the change of the magnetic vector potential. The magnetic potential energy is derived at the expense of current, consuming the power that was induced. What's normal about this is that the motor will not produce an energetic anomaly. What's unusual about this is that you can end up with a situation where there is no sustained current flow in the closed path despite that you would expect an EMF, and/or you can end up with an unusually large capacitance in the coil.
** If it comes from electrical potential energy, then it could mean that the electrical potential energy of charged particles (whether like or opposite pairs) is being consumed even though no battery or capacitor was added to the circuit. What is unusual about this is the source of the electrical potential energy [q Φ].
** If it comes from anything else, well that is already unusual.

My [i.e. kmarinas86's] prediction is that the only kinetic energy consumed would be that corresponding to displacement of the magnetic fields of each charged particle against the magnetic field of the bar magnet, which occurs at the same velocity as the drift velocity of the charge, whereas an anomalous electrical potential energy (in the absence of "attached" batteries and capacitors) will be consumed as a result of the magnetic flux (in units of webers) that is induced at the short (2" or 5 cm) edges "skipping" distance as it is conveyed from one free conduction electron to the next. One can imagine the energy of current "hopping" from one free conduction electron to another just like energy in an "engineered" capacitor where energy hops from electrons on one plate to electrons on another plate. The premise is that displacement current does not produce the same magnetic field as an equivalent conduction current, and any observed magnetic field between the plates is due to conduction current leading to and through the plates as it is being charged or discharged, as opposed to a "displacement current" literally flowing from one plate to another. Look up "Bill Miller" "displacement current": https://www.google.com/search?q="bill+miller"+"displacement+current"

Conclusion: Save up money to build an "S-Motor" (Discovered by kmarinas86 [me] this past summer [either July or August 2014] with these details resolved as of November 26, 2014).
« Last Edit: November 30, 2014, 11:17:21 AM by kmarinas86 »